What is the arc-length parameterization for a given vector function?

Click For Summary
The discussion focuses on finding the arc-length parameterization for the vector function r(t) = <e^(2t), e^(-2t), 2√2 t> for t ≥ 0. Participants analyze the derivative of the function, |r'(t)|, and its integral to determine the arc length s(t). A mistake is identified in the integration step, where it was incorrectly stated that the integral equals zero; it is clarified that the integral is positive due to the nature of the integrand. The conversation also touches on the relationship between the expression and hyperbolic functions, specifically noting that |r'(t)| can be expressed in terms of cosh and sinh functions. The discussion concludes with the implication that hyperbolic functions may be necessary for solving the problem effectively.
Sho Kano
Messages
372
Reaction score
3

Homework Statement


Find the arc-length parameterization for r(t)=\left&lt; { e }^{ 2t },{ e }^{ -2t },2\sqrt { 2 } t \right&gt; ,t\ge 0

Homework Equations


s(t)=\int { \left| \dot { r } (t) \right| dt }

The Attempt at a Solution


\dot { r } (t)=\left&lt; { 2e }^{ 2t },-2{ e }^{ -2t },2\sqrt { 2 } \right&gt; \\ \left| \dot { r } (t) \right| =\sqrt { 4{ e }^{ 4t }+4{ e }^{ -4t }+8 } \\ =2\sqrt { { e }^{ 4t }+{ e }^{ -4t }+2 } \\ =2\sqrt { { e }^{ 4t }+\frac { 1 }{ { e }^{ 4t } } +2 } \\ =2\sqrt { \frac { { e }^{ 8t }+{ 2e }^{ 4t }+1 }{ { e }^{ 4t } } } =2\sqrt { \frac { \left( { e }^{ 4t }+1 \right) \left( { e }^{ 4t }+1 \right) }{ { e }^{ 4t } } } \\ =\frac { 2 }{ { e }^{ 2t } } \left( { e }^{ 4t }+1 \right) \\ ={ 2e }^{ 2t }+{ 2e }^{ -2t }\\ s(t)-s(0)=\int _{ 0 }^{ t }{ { 2e }^{ 2t }+{ 2e }^{ -2t }dt } =0 It can't equal 0...where did I go wrong?
 
Physics news on Phys.org
It looks correct to me until the very last step where you say the integral is zero. The integral is not zero. It couldn't be, because both terms of the integrand are positive.

Note that ##2e^{2t}+2e^{-2t}## is ##2\cosh 2t##. Have a look at the graph of the cosh function and it will be immediately apparent that its definite integral from 0 to ##t## is positive.
 
  • Like
Likes Sho Kano
andrewkirk said:
It looks correct to me until the very last step where you say the integral is zero. The integral is not zero. It couldn't be, because both terms of the integrand are positive.

Note that ##2e^{2t}+2e^{-2t}## is ##2\cosh 2t##. Have a look at the graph of the cosh function and it will be immediately apparent that its definite integral from 0 to ##t## is positive.
Shoot, I made a mistake in the integration. It should be s(t)=2\int { { e }^{ 2t }+{ e }^{ -2t } } =2\left( \frac { { e }^{ 2t } }{ 2 } -\frac { { e }^{ -2t } }{ 2 } \right). That's obviously not zero.
 
What's the next step? Taking the natural log doesn't seem like it will work out, because I'd be left with { e }^{ 4t }-1 inside the log. s={ e }^{ 2t }-{ e }^{ -2t }=\frac { { e }^{ 4t }-1 }{ { e }^{ 2t } }
 
Sho Kano said:

Homework Statement


Find the arc-length parameterization for r(t)=\left&lt; { e }^{ 2t },{ e }^{ -2t },2\sqrt { 2 } t \right&gt; ,t\ge 0

Homework Equations


s(t)=\int { \left| \dot { r } (t) \right| dt }

The Attempt at a Solution


\dot { r } (t)=\left&lt; { 2e }^{ 2t },-2{ e }^{ -2t },2\sqrt { 2 } \right&gt; \\ \left| \dot { r } (t) \right| =\sqrt { 4{ e }^{ 4t }+4{ e }^{ -4t }+8 } \\ =2\sqrt { { e }^{ 4t }+{ e }^{ -4t }+2 } \\ =2\sqrt { { e }^{ 4t }+\frac { 1 }{ { e }^{ 4t } } +2 } \\ =2\sqrt { \frac { { e }^{ 8t }+{ 2e }^{ 4t }+1 }{ { e }^{ 4t } } } =2\sqrt { \frac { \left( { e }^{ 4t }+1 \right) \left( { e }^{ 4t }+1 \right) }{ { e }^{ 4t } } } \\ =\frac { 2 }{ { e }^{ 2t } } \left( { e }^{ 4t }+1 \right) \\ ={ 2e }^{ 2t }+{ 2e }^{ -2t }\\ s(t)-s(0)=\int _{ 0 }^{ t }{ { 2e }^{ 2t }+{ 2e }^{ -2t }dt } =0 It can't equal 0...where did I go wrong?

You could also put ##|\dot{r}(t)| = \sqrt{8} \sqrt{1+\cosh(4t)}##.
 
  • Like
Likes Sho Kano
OK, this problem was a pain
s={ e }^{ 2t }-{ e }^{ -2t }=2sinh(2t)\\ \frac { s }{ 2 } =sinh(2t)\\ arcsinh\left( \frac { s }{ 2 } \right) =2t=ln\left[ \frac { s }{ 2 } +\sqrt { 1+\frac { { s }^{ 2 } }{ 2 } } \right] \\ r(t(s)). So is it unsolvable without using hyperbolic functions?
 
Question: A clock's minute hand has length 4 and its hour hand has length 3. What is the distance between the tips at the moment when it is increasing most rapidly?(Putnam Exam Question) Answer: Making assumption that both the hands moves at constant angular velocities, the answer is ## \sqrt{7} .## But don't you think this assumption is somewhat doubtful and wrong?

Similar threads

Replies
3
Views
2K
  • · Replies 10 ·
Replies
10
Views
1K
  • · Replies 2 ·
Replies
2
Views
1K
  • · Replies 2 ·
Replies
2
Views
4K
  • · Replies 4 ·
Replies
4
Views
3K
Replies
11
Views
2K
  • · Replies 7 ·
Replies
7
Views
3K
  • · Replies 1 ·
Replies
1
Views
2K
Replies
10
Views
2K
  • · Replies 20 ·
Replies
20
Views
3K